LSAT and Law School Admissions Forum

Get expert LSAT preparation and law school admissions advice from PowerScore Test Preparation.

 Adam Tyson
PowerScore Staff
  • PowerScore Staff
  • Posts: 5153
  • Joined: Apr 14, 2011
|
#49914
Although this argument is causal rather than conditional, we can still use some conditional analysis on it. Answer C can be viewed as a Mistaken Negation, akanshalsat. The author argues that the spread of literacy before there is comprehensive education can help topple benign regimes. Perhaps look at that as this diagram:

Literacy + Comprehensive Education :arrow: Topple Benign Regime

Answer C is:

Comprehensive Education :arrow: :arrow: Topple Benign Regime

That alone should be enough to kill it, because a conditional claim does not entail that the author assume a Mistaken Negation of that claim. But, let's apply the Assumption Negation Technique to it. Answer C, negated, would be something like "a comprehensive system of general might not be sufficient to preserve a benign regime." In other words, even if you have that system of education, a benign regime could fall. What does that do to the claim that without that system, benign systems might fall? Nothing! Both could be true - benign systems might be doomed either way, and it could still be that literacy without education makes the fall more likely or faster.

When faced with a conditional claim, the author assumes the contrapositive is true, but he need not make any assumptions about mistaken negations or mistaken reversals, and in fact those might actually hurt his argument.

I hope that makes it clearer!
 lemoncurd
  • Posts: 1
  • Joined: Jan 16, 2019
|
#62007
Thanks for the above explanations! I thought answer choice D's language was too strong, i.e., that it was too much of a stretch to read "illegitimate" vs. "legitimate" reform from the historian's language. How can I distinguish between answer choices that are too extreme in language and not?

Thank you.
 Malila Robinson
PowerScore Staff
  • PowerScore Staff
  • Posts: 296
  • Joined: Feb 01, 2018
|
#62025
Hi lemoncurd,
In this particular question legitimate reformers relates to the folks in the stimulus who have been called true reformers. The illegitimate ones are the ones who are called mere opportunists in the stimulus. So we are matching terms in order to see that this is not too extreme.
However, the more general answer would be to use the assumption negation technique for this question (since it is an assumption question). If you negate answer D then a general lack of education would not necessarily inhibit the ability to differentiate...(etc.). And if that were the case then the second, and after that the third, sentences in the stimulus would not necessarily be true. This means that negating Answer D has harmed the argument, and that makes it the correct choice.
Hope that helps,
Malila
User avatar
 Adam354
  • Posts: 29
  • Joined: Feb 08, 2022
|
#93873
This one confused me a bit because I missed "in the right circumstances."

Literacy "in the right circumstances" can lead to less vulnerability to demagogues.
The "wrong circumstances" is if literacy emerged before general education, which could topple a benign regime.

C) General education if includes literacy, would protect against demagogues.
However, it says "any system of general education," and therefore we cannot assume it would include literacy.

If we removed "in the right circumstances" then we could have presumed that literacy does not necessarily require general education to identify a demagogue, and therefore the danger would be that in the "interim," the demagogue became literate before the general population did, and used that knowledge to manipulate people.

That would still be a stretch though, and enough of a stretch to hopefully go read the stimulus for something missed.

In the right circumstances seems to change everything with this question.
 Adam Tyson
PowerScore Staff
  • PowerScore Staff
  • Posts: 5153
  • Joined: Apr 14, 2011
|
#93907
There's more to it than just "in the right circumstances," Adam354. Note that throughout the stimulus the author uses words and phrases that limit their statements and prevent them from overreaching. For example, saying that the populace is "vulnerable" does not mean that they must, inevitably, fall prey to demagogues. And the author says only that some regimes "may" be toppled, rather than that they will be.

All of this hedging on the part of the author means that we cannot select an answer that is 100% certain on these points, and we should instead be looking for an answer that is about what could occur or what might be true. Answer C is also wrong because the author doesn't have to assume anything about "any" such system, but only about some systems.
User avatar
 teddykim100
  • Posts: 43
  • Joined: Jan 10, 2022
|
#98614
Hello,

I feel like I've come across a concept that describes why answer (C) is wrong. For example, it sort of strikes a similar tone with the "reversing logic" concept (which we are taught is invalid) because we can't say that the author stating lack of education topples regimes, means presence of education preserves regimes (aka, we can't say A=>B, necessitates that B=>A)

What concept/topic would this be? The only thing I can think of is that it is a demonstration of faultily negating an argument (A=>B does NOT equal -A=> -B)
 Robert Carroll
PowerScore Staff
  • PowerScore Staff
  • Posts: 1787
  • Joined: Dec 06, 2013
|
#98688
teddykim,

Adam's post at the top of this page calls answer choice (C) Mistaken Negation, and note that Mistaken Negation is logically equivalent to Mistaken Reversal. So, I think this can be called Mistaken Reversal.

Robert Carroll
User avatar
 teddykim100
  • Posts: 43
  • Joined: Jan 10, 2022
|
#98703
Thank you Robert.

Get the most out of your LSAT Prep Plus subscription.

Analyze and track your performance with our Testing and Analytics Package.